2009 AMC 8 Problems/Problem 13

Revision as of 16:58, 25 December 2012 by Gina (talk | contribs)

Problem

A three-digit integer contains one of each of the digits $1$, $3$, and $5$. What is the probability that the integer is divisible by $5$?

$\textbf{(A)}\  \frac{1}{6}  \qquad \textbf{(B)}\   \frac{1}{3}  \qquad \textbf{(C)}\   \frac{1}{2}  \qquad \textbf{(D)}\  \frac{2}{3}   \qquad \textbf{(E)}\   \frac{5}{6}$

Solution

The three digit numbers are $135,153,351,315,513,531$. The numbers that end in $5$ are divisible are $5$, and the probability of choosing those numbers is $\boxed{\textbf{(B)}\ \frac13}$.

See Also

2009 AMC 8 (ProblemsAnswer KeyResources)
Preceded by
Problem 12
Followed by
Problem 14
1 2 3 4 5 6 7 8 9 10 11 12 13 14 15 16 17 18 19 20 21 22 23 24 25
All AJHSME/AMC 8 Problems and Solutions